Đến nội dung

Senju Hashirama

Senju Hashirama

Đăng ký: 10-05-2015
Offline Đăng nhập: 02-02-2018 - 20:13
-----

#663285 Cho $x, y, z > 0 \in \mathbb{R}$ thỏa...

Gửi bởi Senju Hashirama trong 28-11-2016 - 16:15

Thế để mình chứng minh Mình sẽ chứng minh theo kiểu <=9/4 nha và bạn sẽ thấy nó đúng 

 

$\sum \frac{1}{1+x^{2}}\leq \frac{9}{4} <=> \sum \frac{x^{2}}{x^{2}+1}\geq \frac{3}{4}$

     Cauchy swarch Và dễ chứng minh được điều này nha 

                   $\frac{(x+y+z)^{2}}{x^{2}+y^{2}+z^{2}+3}\geq \frac{3}{4}$
Vậy VT<=9/4<1+3căn3/4

cách của bạn ngược dấu mất rồi 

 

Lời giải:

 Ta có $LHS=\frac{1}{x+y}\left ( \frac{x+y+2z}{z^{2}+1} \right )+\frac{1}{z^{2}+1}=\frac{1}{z^{2}+1}\left ( 2+ \frac{2z}{x+y}\right )$

   Lại có : $z^{2}+1=\left ( x+z \right )\left ( y+z \right )\leq \frac{\left ( x+y+2z \right )^{2}}{4}\Rightarrow 2(\sqrt{z^{2}+1}-z)\leq x+y$

$\Rightarrow LHS\leq \frac{1}{z^{2}+1}\left ( z\left ( \sqrt{z^{2}+1}+z \right )+2 \right ) =1+\frac{z}{\sqrt{z^{2}+1}}+\frac{1}{z^{2}+1}$

 

Mình nghĩ phải là $\frac{z}{z^{2}+1}$  chứ nhỉ 




#657761 $a^{2}+b^{2}+c^{2}=3$

Gửi bởi Senju Hashirama trong 13-10-2016 - 20:21

Ta có : $\sum \frac{a}{b}\geq \frac{\left ( a+b+c \right )^2}{ab+bc+ca}$

Ta cần chứng minh :$(a+b+c)^3\geq 9(ab+bc+ca)=\sqrt{27\left ( a^2+b^2+c^2 \right )\left ( ab+bc+ca \right )^2}$

Dễ thấy : $\sqrt{27\left ( a^2+b^2+c^2 \right )(ab+bc+ca)^2}\leq \sqrt{\left ( a+b+c \right )^6}=(a+b+c)^3$

 Đẳng thức xảy ra $\Leftrightarrow a=b=c=1$




#657535 Đề chọn đội tuyển học sinh giỏi quốc gia tỉnh Quảng Ninh ngày 1 2016-2017

Gửi bởi Senju Hashirama trong 11-10-2016 - 20:10

Bài 2: 

Gọi $P(x,y)$ là phép thế cho $x,y$ : $f(x^2+2f(y))=2y+f^2(x)$   $(1)$

Đặt $f(0)=a$ . $P(0,y)\Rightarrow f(2f(y))=2y+ a^2$

$P(x,2f\left ( y-\frac{a^{2}}{2} \right ))\Rightarrow f(x^2+4y)=2f(y-\frac{a^2}{2})+f^2(x)$   $(2)$

Thế $x=0$ vào $(2)$ $\Rightarrow f(4y)=2f(y-\frac{a^{2}}{2})+a^2\Rightarrow f(x^2+4y)=f(4y)+f^2(x)-a^2$  $(3)$

Thế $y=0$ vào $(3)$ $\Rightarrow f(x^2)=a+f^2(x)-a^2$

Từ đó $\Rightarrow f(x^2+4y)=f(4y)+f(x^2)-a$

Đặt $g(x)=f(x)-a$. Ta có : $g(x+y)=g(x)+g(y)$ với $x\geq 0$ , $\forall y$

Dễ thấy $g(0)=0$ và $g(x)$ lẻ $\Rightarrow g(x+y)=g(x)+g(y)$ với $\forall x,y$

$P(x,0)\Rightarrow f(x^2+4f(0))=f^2(x)\Rightarrow f(x^2+4f(0))-f(0)=f^2(x)-f(0)\geq -f(0)$

$\Rightarrow g(x)\geq -f(0)$   $\Rightarrow g(x)=ax\Rightarrow f(x)=ax+b$ .

Thế vào $(1)$ $\Rightarrow f(x)=x$




#656250 Đề thi chọn đội tuyển Quốc gia tỉnh Thanh Hóa năm 2016-2017(vòng 2)

Gửi bởi Senju Hashirama trong 01-10-2016 - 19:27

Câu 1 : Thực chất là đề IMO shortlist 2009 câu A2

http://www.artofprob...h355781p1932917




#656134 Đề thi chọn học sinh giỏi quốc gia vòng II thành phố Hà Nội 2016-2017

Gửi bởi Senju Hashirama trong 30-09-2016 - 19:53

$P\left ( x,y \right )$ là phép thế cho $x,y$ : $f\left ( x^{2}-f^{2}(y) \right )=xf(x)+y^2$  $(1)$

1, Ta chứng minh $f(0)=0\Leftrightarrow x=0$

Đặt $-f^2(0)=a$ .$P\left ( 0,0 \right )\Rightarrow f\left ( a \right )=0$

$P(0,a)\Rightarrow f(0)=a^2\Rightarrow f(0)=0\vee f(0)=1$

Xét $f(0)=1$ , $P(-1,0)\Rightarrow f(0)=-f(-1)$ (vô lý ) do $f(0)=1$ và $f(-1)=f(a)=0$

$\Rightarrow f(0)=0.$Do đó $P(x,0)\Rightarrow f(x^2)=xf(x) (*) \Rightarrow$ với $f(y)=0$ , từ $(1)$ $\Leftrightarrow y=0$

2/ Ta chứng minh $f$ toàn ánh.

Từ $(*)$ dễ thấy $f$ là hàm lẻ với $x\neq 0$. Lại có $P(0,x)\Rightarrow f\left ( -f^2 (x)\right )=x^2$ 

Từ đó $\Rightarrow f$ toàn ánh 

3/ Chứng minh $f(x)=-x$

Từ các phép thế trên , ta có $f\left ( x^2-f^2(y) \right )=f(x^2)+f(-f^2(y))$ .Lại có $f$ toàn ánh và hàm lẻ $\Rightarrow f(x+y)=f(x)+f(y)$

Từ $(*)$ $\Rightarrow f((x+1)^2))=(x+1)f(x+1)\Rightarrow f(x^2)+2f(x)+f(1)=x(f(x)+f(1))+f(x)+f(1)\Rightarrow f(x)=xf(1)$

$\Rightarrow f(x)=kx$ với $f(1)=k$. Thế vào $(1)$ dễ thấy $k=-1$. 

Vậy $f(x)=-x$




#656007 $P=\sqrt{ab+a}+\sqrt{bc+b}+\sqrt...

Gửi bởi Senju Hashirama trong 29-09-2016 - 19:46

Ta có : $2ab+2a-a^{2}=a\left ( 2b+2-a \right )\geq 0\Rightarrow \sqrt{2\left ( ab+a \right )}\geq a$

 Tương tự $\Rightarrow \sum \sqrt{ab+a}\geq \frac{a+b+c}{\sqrt{2}}=\sqrt{2}$

 Đẳng thức xảy ra $\Leftrightarrow \left ( a,b,c \right )=\left ( 0,0,2 \right )$ và các hoán vị 




#654484 Đề thi chọn đội tuyển quốc gia tỉnh Quảng Bình

Gửi bởi Senju Hashirama trong 17-09-2016 - 14:02

Bài 5 là Iran TST 2013.

Bài 7 là bài 5 IMO 2009 




#651203 Chứng minh rằng với mọi số thực a,b,c ta có : 2(1+abc)+\sqrt{2(1+a^...

Gửi bởi Senju Hashirama trong 25-08-2016 - 13:49

Bài 2: Ta có : $\left ( a^{2}+b+c \right )\left ( 1+b+c \right )\geq \left ( a+b+c \right )^{2}$  $\Rightarrow \sqrt{\frac{a^{2}}{ a^{2}+b+c}} \leq \frac{a\sqrt{b+c+1}}{a+b+c}$

Tương tự $\Rightarrow LHS\leq \sum \frac{a\sqrt{b+c+1}}{a+b+c}\leq \frac{2\sum ab + 4\sum a}{2\sqrt{3}\left ( a+b+c \right )}\leq \frac{\left ( a+b+c \right )^{2}+6(a+b+c)}{3\sqrt{3}(a+b+c)} \leq \sqrt{3}$

Đẳng thức xảy ra $\Leftrightarrow a=b=c=1$

 

Bài 3: 

Ta có : $\sum \frac{a+b+1}{a+b^{2}+c^{3}}\leq \sum \frac{(a+b+1)\left ( a+1+\frac{1}{c} \right )}{\left ( a+b+c \right )^{2}}$

Ta sẽ chứng minh :

 $\sum \left ( a+b+1 \right )\left ( a+1+\frac{1}{c} \right )\leq \left [ \prod \left ( a+1 \right )+1 \right ]\left ( a+b+c \right )$

 Nhưng đây là đẳng thức với $abc=1$  $\Rightarrow $ ĐPCM

Đẳng thức xảy ra $\Leftrightarrow a=b=c=1$




#649966 $f\left ( f\left ( x \right )+y \right )=f\left...

Gửi bởi Senju Hashirama trong 16-08-2016 - 22:09

Tim tất cả các hàm $f:\mathbb{R}\rightarrow \mathbb{R}$ , thỏa mãn 

 $f\left ( f\left ( x \right )+y \right )=f\left ( x+y \right )+xf(y)-xy-x+1$




#647386 $\left\{\begin{matrix}\sqrt{x+2...

Gửi bởi Senju Hashirama trong 31-07-2016 - 20:15

Từ $(1)$ $\Rightarrow \left ( \sqrt{x+2}-\sqrt{y} \right )\left [ \sqrt{x+2} \left ( \sqrt{x+2}+\sqrt{y} \right )+1 \right ]=0$

$\Rightarrow x+2=y$

Từ đó thế vào $(2)$ giải dễ rồi  :D  :D




#646395 $\sum \frac{x^{2}+y^{2}}{x+...

Gửi bởi Senju Hashirama trong 25-07-2016 - 10:34

Đây là 1 bài toán quen thuộc, mình xin gõ lại lời giải bằng '' yếu tố ít nhất'' của thầy Cẩn  :D  :D  :D

Do BĐT thuần nhất , chuẩn hóa cho $x^{2}+y^{2}+z^{2}=3$ 

BĐT $\Leftrightarrow \sum \frac{\left ( x+y \right )^{2}+\left ( x-y \right )^{2}}{2(x+y)}\geq 3$

Ta có :  $LHS\geq x+y+z+\frac{\left ( x-z \right )^{2}}{x+y+z}=\frac{\left ( x-z \right )^{2}+\left ( y-x \right )\left ( y-z \right )}{x+y+z}+x+y+z+\frac{\left ( x-y \right )\left ( y-z \right )}{x+y+z}$

 =$\frac{9}{2\left ( x+y+z \right )}+\frac{x+y+z}{2}+\frac{\left ( x-y \right )\left (y-z \right )}{x+y+z}\geq 3+\frac{\left ( x-y \right )\left (y-z \right )}{x+y+z}$

Bài toán đc chứng minh nếu ta có : $\left ( x-y \right )\left ( y-z \right )\geq 0$

 Đúng nếu cho $y$ nằm giữa $x,z$

Đẳng thức xảy ra tại $x=y=z=1$




#644865 $\frac{a^m+b^m+c^n}{a^n+b^n+c^n}\ge (...

Gửi bởi Senju Hashirama trong 13-07-2016 - 22:24

Không mất tính tổng quát , giả sử $a\geq b\geq c$   

$\Rightarrow a^{n}\geq b^{n}\geq c^{n}$ và  $a^{m-n}\geq b^{m-n}\geq c^{m-n}$

Áp dụng BĐT $Chebyshev$ , ta có 

$\sum a^{m}\geq \frac{1}{3}\sum a^{n}.\sum a^{m-n}$

Tương tự , ta có :

$\sum a^{m-n}\geq \frac{1}{3}\sum a^{m-n-1}\sum a$

Cứ như vậy ta có ĐPCM 




#644698 $\sum \frac{1}{\sqrt{4a^{2}...

Gửi bởi Senju Hashirama trong 12-07-2016 - 17:56

Áp dụng BĐT $Holder$ ,ta có :

$\left ( \sum \frac{1}{\sqrt{4a^{2}+bc}} \right )^{2}\left [ \sum \left ( b+c \right )^{3}\left ( 4a^{2}+bc\right ) \right ]\geq 8 \left (a+b+c \right )^{3}$

Ta sẽ chứng minh : 

   $8\left ( a+b+c \right )^{5}\geq 16 \sum \left ( b+c \right )^{3}\left ( 4a^{2}+bc \right )$ 

Chuẩn hóa cho $a+b+c=1$ , $q=ab+bc+ca$ , $r=abc$ , ta có :

 BĐT $\Leftrightarrow 1\geq 2\left ( 4q-15r+22qr+q \right )$ 

    $\Leftrightarrow 2r\left ( 15-22q \right )-8q^{2}-2q+1\geq 0$

 

Xét $4q-1\leq 0 \Rightarrow \left ( 1-4q \right )\left ( 2q+1 \right )\geq 0$  (Luôn đúng )

Xét $4q-1\geq 0\Rightarrow LHS\geq \frac{2\left ( 15-22q \right )\left ( 4q-1 \right )}{9}-8q^{2}-2q+1$

                                $= \frac{\left ( 4q-1 \right )\left ( 21-62q \right )}{9}\geq 0$

   Luôn đúng do $q\leq \frac{1}{3}<\frac{21}{62}$

   Đẳng thức xảy ra $\Leftrightarrow a=b=c$ và $a=b,c=0$ và các hoán  vị 




#644560 $Min P=\frac{a^3}{b^2+1}+\frac{b^3...

Gửi bởi Senju Hashirama trong 11-07-2016 - 20:17

Đặt $a=\frac{y}{x},b=\frac{z}{y},c=\frac{x}{z}$

$P=\sum \frac{y^{5}}{x^{3}\left ( z^{2} +y^{2}\right )}\geq \frac{(\sum x^{3})^{2}}{xyz(\sum x^{2}z )+\sum x^{3}y^{3}}$

Lại có : $3xyz\leq \sum x^{3}$

$\sum x^{2}z\leq \sum x^{3}$

$3\sum x^{3}y^{3}\leq \left ( \sum x^{3} \right )^{2}$

$\Rightarrow P\geq \frac{3}{2}$

Đẳng thức xảy ra tại a=b=c=1




#643939 $\sum \frac{a^{2}}{\sqrt{(1...

Gửi bởi Senju Hashirama trong 07-07-2016 - 09:58

APMO 2005  :D